You are on page 1of 6

UNIVERSITY OF NUEVA CACERES – NAGA CITY

COLLEGE OF ENGINEERING AND ARCHITECTURE


MODULE # 1
ENGINEERING MECHANICS - STATICS
STATICS OF RIGID BODIES 2. Compute for the magnitude of the y component of the
resultant.
PRINCIPLES OF STATICS
a. 1.445 kN downward
Statics is the branch of mechanics that is concerned with
b. 1.445 kN upward
the analysis of loads (force and torque, or "moment") acting
c. 1.445 kN rightward
on physical systems that do not experience an acceleration
d. 1.445 leftward
(a=0), but rather, are in static equilibrium with their
3. Compute for the resultant and direction of the
environment.
concurrent forces.
a. 6.295 kN (1st quadrant) & 13.270 from the + x axis
Rigid Body
b. 6.295 kN (2nd quadrant) & 13.270 from the - x axis
Idealization of a Solid Body in which deformation
c. 6.295 kN (3rd quadrant & 13.270 from the + y axis
is neglected.
d. 6.295 kN (4th quadrant) & 13.270 from the – y axis
Force
Changes or tends to change the state of the motion
of the body.
SOLUTION 1:
1. Magnitude
Rx = 25 cos 300 – 10 sin 450 – 20 sin 250
2. Position of line of action
Rx = 6.127 kN (Rightward)
3. Direction
SOLUTION 2:
For forces in equilibrium:
Ry = 25 sin 300 + 10 cos 450 – 20 cos 25
∑Fx = 0
Ry = 1.445 kN (Upward)
∑Fy = 0
∑M = 0
SOLUTION 3:
Components of forces:
2 2
R = √𝑅𝑥 + 𝑅𝑦

R = √6.1272 + 1.4452
1.445
𝜃 = tan−1 (6.127)
Fx = F cosθ 𝜃 = 13.27𝑜
Fy = F sinθ R = 6.295 at 13.270 from the positive X- axis

SITUATION 1 SITUATION 2
Refer to the force diagram below Two pulling machine on opposite banks of a river pull a
barge moving parallel to the banks by means of two
horizontal ropes. The tension on these ropes are 180 kN and
212 kN while the angle between them is 60˚.
4. Find the resultant pull on the barge.
a. 206.2 kN b. 408.11 kN
c. 339.9 kN d. 422.67 kN

5. Find the angle between each ropes and the side of canal
a. 38.8˚ and 21.2˚ b. 25.2˚ and 34.8˚
c. 32.7˚ and 27.3˚ d. 41˚ and 19˚
1. Compute for the magnitude of the x component of the
resultant. SOLUTION 4:
a. 6.127 to the left We can solve for the Rx and Ry of this problem by rotating
the x and y axis.
b. 6.127 to the right
c. 6.127 upward
d. 6.127 downward

__M I J D__
UNIVERSITY OF NUEVA CACERES – NAGA CITY
COLLEGE OF ENGINEERING AND ARCHITECTURE
MODULE # 1
ENGINEERING MECHANICS - STATICS
X intercept & Y intercept
Horizontal and vertical distances where the
resultant intersect our X and Y axis from the origin (0,0)
Varignon’s Theorem
The Moment of the whole is equal to the
summation of the moment by parts
Ry x = F1x1 + F2x2 … + M1 + M2
Rx = 180 + 212 cos 60
Rx = 286 kN
SITUATION 3
Ry = 212 sin 60 In the force system shown below, find:
Ry = 183.597 kN

2 2
R = √𝑅𝑥 + 𝑅𝑦

R = √2862 + 183.5972

R = 339.859 kN

SOLUTION 5:

7. The resultant
a. 52.148 kN b. 57.178 kN
c. 32.516 kN d. 40.956 kN
8. Angle that the resultant makes with the horizontal
a. 43.059o b. 33.407o

183.597 c. 48.143o d. 51.091o


𝜃 = tan−1 ( )
286
9. X – intercept of the resultant
𝑜
𝜃 = 32.7
a. 2.156 m b. 3.626 m
c. 4.177 m d. 6.018 m

PROBLEM 6
Find the tension on the chord AB of the bow if an arrow is SOLUTION 7
pulled at a force of 30 kgs. Rx = 40 cos 100 + 20 (4/5) – 10 cos 400
Rx = 47.731 kN

Ry = 10 sin 40 + 20 + 20 (3/5) – 40 sin 10


Ry = 31.482 kN

2 2
a. 143 N b. 220 N R = √𝑅𝑥 + 𝑅𝑦
c. 177 N d. 152 N
R = √47.731 2 + 31.482 2
SOLUTION 6 R = 57.178 kN
Pull = 30 kgs (9.81 m/s2 ) = 294.3 kN
SOLUTION 8
31.482
∑Fx = 0 𝜃 = tan−1 (47.731)
294.3 = AB cos 45 + AC cos 30
∑Fy = 0 𝜃 = 33.4070
AB sin45 = AC sin 30 SOLUTION 9
3
𝑅𝑦 𝑥̅ = − 20 (5) (6) − 20 (3) − 10 cos 40 (6) +
Solve simultaneously
AB = 152.34 N 40 cos 10 (2) − 15
AC = 215.442 N

__M I J D__
UNIVERSITY OF NUEVA CACERES – NAGA CITY
COLLEGE OF ENGINEERING AND ARCHITECTURE
MODULE # 1
ENGINEERING MECHANICS - STATICS
𝑅𝑦 𝑥̅ = Ʃ 𝑚𝑜𝑚𝑒𝑛𝑡𝑠 Fy = 127.0 N
Fz = -190.5 N
3 ƩFx = 31.14 N
31.482 𝑥̅ = − 20 (5) (6) − 20 (3) − 10 cos 40 (6) +
ƩFy = 203.324 N
40 cos 10 (2) − 15
ƩFz = -37.85 N
𝑥̅ = 3.626

SOLUTION 13
FORCE SYSTEM ON SPACE
Compute the resultant force
SITUATION 4
Given the concurrent forces having the following F = √31.142 + 202.3242 + (−37.85)2

magnitudes and passing through the origin and the F = 208.194 N

indicated points:
A = 400 N @ P1(3, 2, 4) SITUATION 5

B = 300 N @ P2(7, -2, -4) A circular steel plate supported on 3 posts. A, B and C

C = 500 N @ (-2, 7, -3) which are equally spaced along its circumference. A load P

10. Compute for the x component of the resultant = 1350N is at a distance x = 0.5 m from post A along the
a. 203.324 N b. 31.14 N diameter. Diameter of steel plate is 1.8 m.
b. 208.194 N d. -37.85 N 14. Find the reaction at post A due to force P alone.
11. Compute for the y component of the resultant
a. 800 N b. 900 N
a. 203.324 N b. 31.14 N
b. 208.194 N d. -37.85 N c. 850 N d. 950 N
12. Compute for the z component of the resultant 15. Find the reaction at post B due to force P alone.
a. 203.324 N b. 31.14 N a. 200 N b. 250 N
b. 208.194 N d. -37.85 N
c. 230 N d. 280 N
13. Compute for the resultant
a. 203.324 N b. 31.14 N SOLUTION 14
b. 208.194 N d. -37.85 N

SOLUTION 10 - 12
FORCE A
𝐹 𝐹𝑥 𝐹𝑦 𝐹𝑧
= = =
𝑑 ∆𝑥 ∆𝑦 ∆𝑧
400 𝐹𝑥 𝐹𝑦 𝐹𝑧 Solve for x
= = =
√29 (3 − 0) (2 − 0) (4 − 0) 𝑥
cos 60 =
Fx = 222.83 N 0.9
Fy = 148.556 N x = 0.45

Fy = 297.113 N ƩMb&c = 0

FORCE B P (0.45 + 0.4) = A (0.45 + 0.9)

300 𝐹𝑥 𝐹𝑦 𝐹𝑧 1350(0.85) = A (1.35)


= = =
√69 (7 − 0) (−2 − 0) (−4 − 0) A = 850N
Fx = 252.81 N SOLUTION 15
Fy = -72.232 N ƩFy = 0
Fz = -144.463 N 1350 = A + B + C ; B = C
FORCE C 1350 = 850 + 2B
𝐹 𝐹𝑥 𝐹𝑦 𝐹𝑧 B = 250
= = =
𝑑 ∆𝑥 ∆𝑦 ∆𝑧
500 𝐹𝑥 𝐹𝑦 𝐹𝑧
= = = SITUATION 6
√62 (−2 − 0) (7 − 0) (−3 − 0)
A cylinder weighing 400 kN is held against a smooth
𝐹𝑥 = − 444.5 𝑁 inclined surface by means of the weightless rod AB.

__M I J D__
UNIVERSITY OF NUEVA CACERES – NAGA CITY
COLLEGE OF ENGINEERING AND ARCHITECTURE
MODULE # 1
ENGINEERING MECHANICS - STATICS
16. Determine the Force PAB exerted by the cylinder to the Given the force system:
rod and the inclined surface If 𝐹 = 900 𝑁, and 𝑃 = 1200 𝑁.
a. 289.55 lbs b. 109.15 lbs 18. The moment about point A.
a. 868.46 N-m b. 735.33 N-m
c. 378.35 lbs d. 418.61 lbs c. 1085.57 N-m d.1266.29N-m
17. Determine the Force N exerted by the cylinder to the
19. The moment about point B.
inclined surface a. -513.85 N-m b. -481.86N-m
a. 289.55 lbs b. 109.15 lbs c. -467.14 N-m d. -385.49N-m

c. 378.35 lbs d. 418.61 lbs 20. The moment about point C.


a. 63.47 N-m b. 84.60 N-m
c. 79.34 N-m d. 76.91 N-m

21. The moment about point D.


a. 765.37 N-m b. 860.20 N-m
c. 637.81 N-m d. 772.91 N-m

22. The resultant force of the system.


a. 1872.55N b. 1407.93N
c. 1702.31N d. 1759.91N
SOLUTION 16-17
SOLUTION 18
400
𝜃𝑝 = 𝑡𝑎𝑛−1 ( ) = 33.69°
600
400
𝜃𝑓 = 𝑡𝑎𝑛−1 ( ) = 63.43°
200
𝑃𝑥 = 1200𝑐𝑜𝑠33.69 = 998.46
𝑃𝑦 = 1200𝑠𝑖𝑛33.69 = 665.64
𝐹𝑥 = 900𝑐𝑜𝑠63.43 = 402.56
𝐹𝑦 = 900𝑠𝑖𝑛63.43 = 804.95
Σ𝑀𝐴 = 998.46(400) − 665.64(100)
+402.56(400) + 804.95(300)
Σ𝑀𝐴 = 735329 𝑁 − 𝑚𝑚
Σ𝑀𝐴 = 735.33 𝑁 − 𝑚 (answer)

SOLUTION 19
Σ𝑀𝐵 = −998.46(400) − 665.64(100)
−402.56(400) + 804.95(300)
Σ𝑀𝐵 = −385487 𝑁 − 𝑚𝑚
Σ𝑀𝐵 = −385.49 𝑁 − 𝑚 (answer)

By sine law SOLUTION 20


Σ𝑀𝐶 = 665.64(700) − 804.95(500)
400 PAB N
= = Σ𝑀𝐶 = 63473 𝑁 − 𝑚𝑚
sin 60 sin 55 sin 65 Σ𝑀𝐶 = 63.47 𝑁 − 𝑚 (answer)
PAB = 378.35 lbs
SOLUTION 21
N = 418.61 lbs Σ𝑀𝐷 = 998.46(400) + 665.64(600)
+402.56(400) − 804.95(400)
Σ𝑀𝐷 = 637812 𝑁 − 𝑚𝑚
SITUATION 7 Σ𝑀𝐷 = 637.81 𝑁 − 𝑚 (answer)

SOLUTION 22
𝑃𝑥 = 1200𝑐𝑜𝑠33.69 = 998.46
𝑃𝑦 = 1200𝑠𝑖𝑛33.69 = 665.64
𝐹𝑥 = 900𝑐𝑜𝑠63.43 = 402.56
𝐹𝑦 = 900𝑠𝑖𝑛63.43 = 804.95
(998.46 + 402.56)2
𝑅=√
+(665.64 − 804.95)2
𝑅 = 1407.93𝑁 (answer)

SITUATION 8
A suspended girder shown is supported by a series of
hangers, uniformly spaced along a parabolic cable.

__M I J D__
UNIVERSITY OF NUEVA CACERES – NAGA CITY
COLLEGE OF ENGINEERING AND ARCHITECTURE
MODULE # 1
ENGINEERING MECHANICS - STATICS
−𝑅2 (8) + 300(11) − 200(5) + 110(12)(1) = 0
𝑅1 = 452.5 𝑘𝑁 (answer)

SITUATION 10
For the truss loaded as shown:

23. Compute the tension in the cable at mid-span C,


whose slope is zero.
a. 599.59 kN b. 540.18 kN
c. 399.73 kN d. 432.14 kN

24. Compute the vertical reaction at Support A.


a. 275 kN b. 73.33 kN
c. 110 kN d. 220 kN 28. What is the reaction at the hinge support?

25. Determine the resulting sag, “y” if the maximum a. 240 kN b. 200 kN
tension in the cable is 450 kN? c. 160kN d. 320 kN
a. 5.09 m b. 4.24m 29. What is the force in member BD?
c. 4.66 m d. 5.13 m a. 240 kN b. 200 kN
c. 160kN d. 320 kN
SOLUTION 23 30. What is the force in member CD?
Σ𝑀𝐴 = 0
(𝐻)(3.5) = 11(25)(5.5) a. 240 kN b. 200 kN
𝐻 = 432.14𝑘𝑁 (𝑎𝑛𝑠𝑤𝑒𝑟) c. 160kN d. 320 kN
31. What is the force in member CE?
SOLUTION 24 a. 240 kN b. 200 kN
2𝑅 = 25(22) c. 160kN d. 320 kN
𝑅 = 275𝑘𝑁 (𝑎𝑛𝑠𝑤𝑒𝑟)

SOLUTION 25 SOLUTION 28
𝐻2 = (450)2 − (275)2
𝐻 = 356.20𝑘𝑁
Σ𝑀𝐴 = 0
356.20𝑦 = 275(5.5)
𝑦 = 4.24𝑚 (𝑎𝑛𝑠𝑤𝑒𝑟)

SITUATION 9
Given the beam shown in figure; determine the following.

ƩMF = 0
36 R1 = 360 (12)
R1 = 120 kN
26. Determine the reaction R1 for the beam loaded as R1 + R2 = 360
shown in the figure
a. 1709.38 kN b. 1282.35 kN R2 = 240 kN
c. 1367.50 kN d. 1410.59 kN 9
tan 𝜃 =
12
27. Determine the reaction R2 for the beam loaded as
shown in the figure. 𝜃 = 36.80
a. 452.50 kN b. 565.63 kN SOLUTION 29
c. 466.76 kN d. 432.14 kN
ƩMC = 0
SOLUTION 26 BD (9) = 120(12)
Σ𝑀𝑅2 = 0 BD = 160 kN (Compression)
𝑅1 (8) − 200(13) + 300(3) − 110(12)(7) = 0
𝑅1 = 1367.5 𝑘𝑁 (answer) SOLUTION 30 - 31
ƩMD = 0
SOLUTION 27 CE (9) = 120(4)
Σ𝑀𝑅1 = 0
__M I J D__
UNIVERSITY OF NUEVA CACERES – NAGA CITY
COLLEGE OF ENGINEERING AND ARCHITECTURE
MODULE # 1
ENGINEERING MECHANICS - STATICS
CE = 320 kN (Tension)
CD sin 36.80 = 120
CD = 200 kN (Compression)

SITUATION 11
A 250 kg block is resting on a rough horizontal surface for 𝑊 = 250𝑘𝑔 𝑥 9.81 = 2.45𝑘𝑁
which the coefficient of friction is 0.35. Σ𝐹𝑣 = 0
32. Determine the horizontal force P that is necessary to 𝑁 = 2.45 − 𝑃𝑠𝑖𝑛𝛽
move the block 𝑓 = 𝜇𝑁 = (0.35)(2.45 − 𝑃𝑠𝑖𝑛𝛽)
a. 1.0719 kN b. 0.8575 kN 𝑓 = 0.8575 − 0.35𝑃𝑠𝑖𝑛𝛽
c. 0.9744 kN d. 0.8947 kN Σ𝐹𝐻 = 0
𝑃𝑐𝑜𝑠𝛽 = 0.8575 − 0.35𝑃𝑠𝑖𝑛𝛽
0.8575 = 𝑃𝑐𝑜𝑠𝛽 + 0.35𝑃𝑠𝑖𝑛𝛽
33. Determine the force P required to cause motion to 0.8575 = 𝑃(𝑐𝑜𝑠𝛽 + 0.35𝑠𝑖𝑛𝛽)
impend if applied to the block downward at 30° with the 0.8575
horizontal. 𝑃=
(𝑐𝑜𝑠𝛽 + 0.35𝑠𝑖𝑛𝛽)
a. 1.24 kN b. 1.71 kN To minimize the force P, we have to differentiate then equate
c. 1.56kN d. 1.69 kN to zero
𝒅𝑷 −𝟎. 𝟖𝟓𝟕𝟓(−𝒔𝒊𝒏𝜷 + 𝟎. 𝟑𝟓𝒄𝒐𝒔𝜷)
34. What minimum force is required to start motion? = =𝟎
𝒅𝜷 (𝑐𝑜𝑠𝛽 + 0.35𝑠𝑖𝑛𝛽)2
a. 0.8903 kN b. 0.7285 kN 𝜷 = 𝟏𝟗. 𝟐𝟗°
c. 0.7358 kN d. 0.8094 kN Minimum value of P
0.8575
𝑃=
(cos 19.29 + 0.35 sin 19.29)
SOLUTION 31 𝑷𝒎𝒊𝒏 = 𝟎. 𝟖𝟎𝟗𝟒 𝒌𝑵 (𝒂𝒏𝒔𝒘𝒆𝒓)
Force is applied horizontally

𝑊 = 250𝑘𝑔 𝑥 9.81 = 2.45𝑘𝑁


Σ𝐹𝑣 = 0
𝑁 = 2.45 𝑘𝑁
𝑓 = 𝜇𝑁 = (0.35)(2.45)
𝑓 = 0.8575 𝑘𝑁
Σ𝐹𝐻 = 0
𝑃=𝑓
𝑃 = 0.8575 𝑘𝑁 (𝑎𝑛𝑠𝑤𝑒𝑟)

SOLUTION 32
Downward force at 30o from the horizontal

𝑊 = 250𝑘𝑔 𝑥 9.81 = 2.45𝑘𝑁


Σ𝐹𝑣 = 0
𝑁 = 2.45 + 𝑃𝑠𝑖𝑛30
𝑓 = 𝜇𝑁 = (0.35)(2.45 + 𝑃𝑠𝑖𝑛30)
𝑓 = 0.8575 + 0.175𝑃
Σ𝐹𝐻 = 0
𝑃𝑐𝑜𝑠30 = 0.8575 + 0.175𝑃
𝑃 = 1.24 𝑘𝑁 (𝑎𝑛𝑠𝑤𝑒𝑟)

SOLUTION 33
Minimum force required to cause impending motion

__M I J D__

You might also like